Difference between revisions of "2012 AMC 10B Problems/Problem 19"
(→Problem) |
|||
Line 3: | Line 3: | ||
<math>\textbf{(A)}\ \frac{133}{2}\qquad\textbf{(B)}\ 67\qquad\textbf{(C)}\ \frac{135}{2}\qquad\textbf{(D)}\ 68\qquad\textbf{(E)}\ \frac{137}{2}</math> | <math>\textbf{(A)}\ \frac{133}{2}\qquad\textbf{(B)}\ 67\qquad\textbf{(C)}\ \frac{135}{2}\qquad\textbf{(D)}\ 68\qquad\textbf{(E)}\ \frac{137}{2}</math> | ||
− | + | [[Category: Introductory Geometry Problems]] | |
==Solution== | ==Solution== |
Revision as of 10:47, 13 August 2014
Problem
In rectangle , , , and is the midpoint of . Segment is extended 2 units beyond to point , and is the intersection of and . What is the area of ?
Solution
The easiest way to find the area would be to find the area of and subtract the areas of and You can easily get the area of because you know and , so 's area is . However, for triangle you don't know However, you can note that triangle is similar to triangle through AA. You see that So, You can do for and Now, you can find the area of which is Now, you do which makes the answer (C).
See Also
2012 AMC 10B (Problems • Answer Key • Resources) | ||
Preceded by Problem 18 |
Followed by Problem 20 | |
1 • 2 • 3 • 4 • 5 • 6 • 7 • 8 • 9 • 10 • 11 • 12 • 13 • 14 • 15 • 16 • 17 • 18 • 19 • 20 • 21 • 22 • 23 • 24 • 25 | ||
All AMC 10 Problems and Solutions |
The problems on this page are copyrighted by the Mathematical Association of America's American Mathematics Competitions.